- UID
- 779686
- 在线时间
- 小时
- 注册时间
- 2012-7-11
- 最后登录
- 1970-1-1
- 主题
- 帖子
- 性别
- 保密
|
4.
Medical researcher: As expected, records covering the last four years of ten major hospitals indicate that babies born prematurely were more likely to have low birth weights and to suffer from health problems than were babies not born prematurely. These records also indicate that mothers who had received adequate prenatal care were less likely to have low birth weight babies than were mothers who had received inadequate prenatal care. Adequate prenatal care, therefore, significantly decreases the risk of low birth weight babies.
In this argument, there is more than one logical line, thus making it difficult to understand. Firstly, the researcher points out babies born prematurely are more likely to have low birth weights than babies not born prematurely. Secondly, the argument reasons that since mothers who have received inadequate prenatal care are more likely to have low birth weight babies than mothers who have received adequate prenatal care, adequate prenatal care, therefore, significantly decreases the risk of low birth weight babies. How to weaken the argument? Since the conclusion is about relationship between adequate prenatal care and the risk of low birth weight babies, if we can break the relationship, we’ll solve the problem. So the answer should refer to adequate prenatal care and low birth weight babies.
这道题和OG里面有一道关于结婚和长寿关系的题非常类似,其中也引入了如吸烟等不良习惯等因素使得逻辑关系比较复杂。但是,需要注意结论是什么,把相关关系找出来,从而逐一击破! Which one of the following, if true, most weakens the medical researcher’s argument?
(A) The hospital records indicate that many babies that are born with normal birth weights are born to mothers who had inadequate prenatal care.
It is normally that some mothers who had inadequate prenatal care would have babies with normal birth weights, thus not weakening the argument. (B) Mothers giving birth prematurely are routinely classified by hospitals as having received inadequate prenatal care when the record of that care is not available.
Firstly, the answer choice connects low birth weight babies, whose mothers gave birth prematurely, with prenatal care. So it seems to be valid. What’s more, the answer choice points out that in this case, the hospitals increase the mothers who received inadequate prenatal by adding mothers giving birth prematurely when the record of that care is not available. Thus, the real situation may be that not so much inadequate prenatal care contributes to the low birth weight babies. BA
This answer choice falls in the category of “Showing a statistical problem exists with the data uses to make the causal statement.” (C) The hospital records indicate that low birth weight babies were routinely classified as having been born prematurely.
At first glance, it seems like a contender. But the choice actually has no effect on the conclusion, which refers to prenatal care and low birth weight babies. (D) Some babies not born prematurely, whose mothers received adequate prenatal care, have low birth weights.
The situation is similar to A. (E) Women who receive adequate prenatal care are less likely to give birth prematurely than are women who do not receive adequate prenatal care.
Actually, this choice strengthens the argument, rather than weakens.
65. (35041-!-item-!-188;#058&007595) Sales of telephones have increased dramatically over the last year. In order to take advantage of this increase, Mammoth Industries plans to expand production of its own model of telephone, while continuing its already very extensive advertising of this product. Which of the following, if true, provides most support for the view that Mammoth Industries cannot increase its sales of telephones by adopting the plan outlined above?
(A) Although it sells all of the telephones that it produces, Mammoth Industries' share of all telephone sales has declined over the last year.
It seems to be contender. However, even though the Mammoth Industries’ share has declined over the last year, the total sales of MI can increase. (B) Mammoth Industries' average inventory of telephones awaiting shipment to retailers has declined slightly over the last year.
Actually, since MI’s average inventory declined slightly, it actually strengthens the argument. (C) Advertising has made the brand name of Mammoth Industries' telephones widely known, but few consumers know that Mammoth Industries owns this brand.
It does point out a weakness in the advertising, but it has no relationship with the sales increase. I wonder a better explanation.
A more thorough analysis reveals that the answer choice is irrelevant, because the fact whether consumers know that MI owns this brand, considering MI’s telephones widely known, has no effect on the argument. (D) Mammoth Industries' telephone is one of three brands of telephone that have together accounted for the bulk of the last year's increase in sales.
It cannot weaken the argument. (E) Despite a slight decline in the retail price, sales of Mammoth Industries' telephones have fallen in the last year.
Yes, it makes clearly that although the price declined, sales of MI’s telephones still fell in last year. BA
66. (35089-!-item-!-188;#058&007596) Bank depositors in the United States are all financially protected against bank failure because the government insures all individuals' bank deposits. An economist argues that this insurance is partly responsible for the high rate of bank failures, since it removes from depositors any financial incentive to find out whether the bank that holds their money is secure against failure. If depositors were more selective, then banks would need to be secure in order to compete for depositors' money. The economist's argument makes which of the following assumptions?
(A) Bank failures are caused when big borrowers default on loan repayments.
Although the argument points out that the government insure is partly responsible for the high rate of bank failures, the reason of bank failures is actually irrelevant. (B) A significant proportion of depositors maintain accounts at several different banks.
The data has no effect on the argument. (C) The more a depositor has to deposit, the more careful he or she tends to be in selecting a bank.
This is true, but we do not know the ability of depositors to choose the secure bank. (D) The difference in the interest rates paid to depositors by different banks is not a significant factor in bank failures.
The same to A. (E) Potential depositors are able to determine which banks are secure against failure.
Yes, in this case, depositors become more selective, and will make a more reasonable choice, thus forcing banks to compete for depositors’ money. On the other hand, if we use “negation”, we’ll find that the failure to determine which banks are secure against failure will make the conclusion fallacious. BA
67.前面的作业已经出现过了
68. (35185-!-item-!-188;#058&007601) Manufacturers sometimes discount the price of a product to retailers for a promotion period when the product is advertised to consumers. Such promotions often result in a dramatic increase in amount of product sold by the manufacturers to retailers. Nevertheless, the manufacturers could often make more profit by not holding the promotions. Which of the following, if true, most strongly supports the claim above about the manufacturers' profit?
(A) The amount of discount generally offered by manufacturers to retailers is carefully calculated to represent the minimum needed to draw consumers' attention to the product.
Since the minimum needed to draw consumers’ attention to the product, we may think that if the amount of discount increases slightly, the sales would increase slightly, thus weakening the argument. (B) For many consumer products the period of advertising discounted prices to consumers is about a week, not sufficiently long for consumers to become used to the sale price.
It seems to be a contender, but a more thorough analysis reveals that it is a Shell Game answer. The argument refers to whether discount causes more profit, having no relationship with consumers who become used to the sale price. Even though a week is not sufficiently long for consumers, the sales could still increase, thus increasing total profit. (C) For products that are not newly introduced, the purpose of such promotions is to keep the products in the minds of consumers and to attract consumers who are currently using competing products.
It still points out the advantage of the promotion, thus not weakening the argument. (D) During such a promotion retailers tend to accumulate in their warehouses inventory bought at discount; they then sell much of it later at their regular price.
In this case, we can see that such promotions do not cause consumers to buy more products, but cause retailers to buy more. The real benefits are obtained by retailers, rather than manufactures. So there is no need to make such promotions. BA (E) If a manufacturer fails to offer such promotions but its competitor offers them, that competitor will tend to attract consumers away from the manufacturer's product.
In fact, this answer choice weakens the argument. |
|